Can an uncountable group have a countable number of subgroups?











up vote
2
down vote

favorite
1













Can an uncountable group have only a countable number of subgroups?



Please give examples if any exist!




Edit: I want a group having uncountable cardinality but having a countable number of subgroups.



By countable number of subgroups, I mean the collection of all subgroups of a group is countable.










share|cite|improve this question




















  • 1




    I'm frankly a bit surprised at the negative reaction to this question.
    – Noah Schweber
    1 hour ago















up vote
2
down vote

favorite
1













Can an uncountable group have only a countable number of subgroups?



Please give examples if any exist!




Edit: I want a group having uncountable cardinality but having a countable number of subgroups.



By countable number of subgroups, I mean the collection of all subgroups of a group is countable.










share|cite|improve this question




















  • 1




    I'm frankly a bit surprised at the negative reaction to this question.
    – Noah Schweber
    1 hour ago













up vote
2
down vote

favorite
1









up vote
2
down vote

favorite
1






1






Can an uncountable group have only a countable number of subgroups?



Please give examples if any exist!




Edit: I want a group having uncountable cardinality but having a countable number of subgroups.



By countable number of subgroups, I mean the collection of all subgroups of a group is countable.










share|cite|improve this question
















Can an uncountable group have only a countable number of subgroups?



Please give examples if any exist!




Edit: I want a group having uncountable cardinality but having a countable number of subgroups.



By countable number of subgroups, I mean the collection of all subgroups of a group is countable.







group-theory examples-counterexamples infinite-groups






share|cite|improve this question















share|cite|improve this question













share|cite|improve this question




share|cite|improve this question








edited 1 hour ago









Shaun

8,074113577




8,074113577










asked 1 hour ago









Cloud JR

731416




731416








  • 1




    I'm frankly a bit surprised at the negative reaction to this question.
    – Noah Schweber
    1 hour ago














  • 1




    I'm frankly a bit surprised at the negative reaction to this question.
    – Noah Schweber
    1 hour ago








1




1




I'm frankly a bit surprised at the negative reaction to this question.
– Noah Schweber
1 hour ago




I'm frankly a bit surprised at the negative reaction to this question.
– Noah Schweber
1 hour ago










2 Answers
2






active

oldest

votes

















up vote
8
down vote













No. Suppose $G$ is an uncountable group. Every element $g$ of $G$ belongs to a countable subgroup of $G$, namely the cyclic subgroup $langle grangle$. Thus $G$ is the union of all of its countable subgroups. Since a countable union of countable sets is countable, $G$ must have uncountably many countable subgroups.






share|cite|improve this answer




























    up vote
    4
    down vote













    EDIT: bof's answer is the right one, but the construction below - while completely pointless overkill - is still an example of a useful technique, so I'm leaving this answer up.





    No, this cannot happen.



    Suppose $G$ is a group and $A$ is a countable subset of $G$. Then the closure of $A$ under the group operations ($*$ and $^{-1}$) of $G$, $langle Arangle$, is again countable - this is a good exercise (HINT: the set of finite strings from a countable set is countable).



    With this in hand, if $G$ is an uncountable group we can build an uncountable chain of subgroups of $G$, as follows:




    • We will define a countable subgroup $A_delta$ for every countable ordinal $delta$. There are uncountably many of these, so if we can do this we'll be done.


    • We let $A_0$ be the trivial subgroup.


    • Having defined $A_eta$ for every $eta<delta$, we let $a$ be some element of $G$ not in $bigcup_{eta<delta}A_eta$ - which exists, since this is a countable union of countable subgroups, and $G$ is uncountable - and let $A_delta=langle (bigcup_{eta<delta}A_eta)cup{a}rangle$.


    • It's easy to prove by transfinite induction that $(A_delta)_{delta<omega_1}$ is a strictly increasing chain of countable subgroups of $G$, so we're done.







    share|cite|improve this answer





















    • Both answers use the axiom of choice. In ZF, can there be an uncountable group with only a countable number of subgroups?
      – bof
      54 mins ago











    Your Answer





    StackExchange.ifUsing("editor", function () {
    return StackExchange.using("mathjaxEditing", function () {
    StackExchange.MarkdownEditor.creationCallbacks.add(function (editor, postfix) {
    StackExchange.mathjaxEditing.prepareWmdForMathJax(editor, postfix, [["$", "$"], ["\\(","\\)"]]);
    });
    });
    }, "mathjax-editing");

    StackExchange.ready(function() {
    var channelOptions = {
    tags: "".split(" "),
    id: "69"
    };
    initTagRenderer("".split(" "), "".split(" "), channelOptions);

    StackExchange.using("externalEditor", function() {
    // Have to fire editor after snippets, if snippets enabled
    if (StackExchange.settings.snippets.snippetsEnabled) {
    StackExchange.using("snippets", function() {
    createEditor();
    });
    }
    else {
    createEditor();
    }
    });

    function createEditor() {
    StackExchange.prepareEditor({
    heartbeatType: 'answer',
    convertImagesToLinks: true,
    noModals: true,
    showLowRepImageUploadWarning: true,
    reputationToPostImages: 10,
    bindNavPrevention: true,
    postfix: "",
    imageUploader: {
    brandingHtml: "Powered by u003ca class="icon-imgur-white" href="https://imgur.com/"u003eu003c/au003e",
    contentPolicyHtml: "User contributions licensed under u003ca href="https://creativecommons.org/licenses/by-sa/3.0/"u003ecc by-sa 3.0 with attribution requiredu003c/au003e u003ca href="https://stackoverflow.com/legal/content-policy"u003e(content policy)u003c/au003e",
    allowUrls: true
    },
    noCode: true, onDemand: true,
    discardSelector: ".discard-answer"
    ,immediatelyShowMarkdownHelp:true
    });


    }
    });














    draft saved

    draft discarded


















    StackExchange.ready(
    function () {
    StackExchange.openid.initPostLogin('.new-post-login', 'https%3a%2f%2fmath.stackexchange.com%2fquestions%2f3031937%2fcan-an-uncountable-group-have-a-countable-number-of-subgroups%23new-answer', 'question_page');
    }
    );

    Post as a guest















    Required, but never shown

























    2 Answers
    2






    active

    oldest

    votes








    2 Answers
    2






    active

    oldest

    votes









    active

    oldest

    votes






    active

    oldest

    votes








    up vote
    8
    down vote













    No. Suppose $G$ is an uncountable group. Every element $g$ of $G$ belongs to a countable subgroup of $G$, namely the cyclic subgroup $langle grangle$. Thus $G$ is the union of all of its countable subgroups. Since a countable union of countable sets is countable, $G$ must have uncountably many countable subgroups.






    share|cite|improve this answer

























      up vote
      8
      down vote













      No. Suppose $G$ is an uncountable group. Every element $g$ of $G$ belongs to a countable subgroup of $G$, namely the cyclic subgroup $langle grangle$. Thus $G$ is the union of all of its countable subgroups. Since a countable union of countable sets is countable, $G$ must have uncountably many countable subgroups.






      share|cite|improve this answer























        up vote
        8
        down vote










        up vote
        8
        down vote









        No. Suppose $G$ is an uncountable group. Every element $g$ of $G$ belongs to a countable subgroup of $G$, namely the cyclic subgroup $langle grangle$. Thus $G$ is the union of all of its countable subgroups. Since a countable union of countable sets is countable, $G$ must have uncountably many countable subgroups.






        share|cite|improve this answer












        No. Suppose $G$ is an uncountable group. Every element $g$ of $G$ belongs to a countable subgroup of $G$, namely the cyclic subgroup $langle grangle$. Thus $G$ is the union of all of its countable subgroups. Since a countable union of countable sets is countable, $G$ must have uncountably many countable subgroups.







        share|cite|improve this answer












        share|cite|improve this answer



        share|cite|improve this answer










        answered 1 hour ago









        bof

        49.5k455118




        49.5k455118






















            up vote
            4
            down vote













            EDIT: bof's answer is the right one, but the construction below - while completely pointless overkill - is still an example of a useful technique, so I'm leaving this answer up.





            No, this cannot happen.



            Suppose $G$ is a group and $A$ is a countable subset of $G$. Then the closure of $A$ under the group operations ($*$ and $^{-1}$) of $G$, $langle Arangle$, is again countable - this is a good exercise (HINT: the set of finite strings from a countable set is countable).



            With this in hand, if $G$ is an uncountable group we can build an uncountable chain of subgroups of $G$, as follows:




            • We will define a countable subgroup $A_delta$ for every countable ordinal $delta$. There are uncountably many of these, so if we can do this we'll be done.


            • We let $A_0$ be the trivial subgroup.


            • Having defined $A_eta$ for every $eta<delta$, we let $a$ be some element of $G$ not in $bigcup_{eta<delta}A_eta$ - which exists, since this is a countable union of countable subgroups, and $G$ is uncountable - and let $A_delta=langle (bigcup_{eta<delta}A_eta)cup{a}rangle$.


            • It's easy to prove by transfinite induction that $(A_delta)_{delta<omega_1}$ is a strictly increasing chain of countable subgroups of $G$, so we're done.







            share|cite|improve this answer





















            • Both answers use the axiom of choice. In ZF, can there be an uncountable group with only a countable number of subgroups?
              – bof
              54 mins ago















            up vote
            4
            down vote













            EDIT: bof's answer is the right one, but the construction below - while completely pointless overkill - is still an example of a useful technique, so I'm leaving this answer up.





            No, this cannot happen.



            Suppose $G$ is a group and $A$ is a countable subset of $G$. Then the closure of $A$ under the group operations ($*$ and $^{-1}$) of $G$, $langle Arangle$, is again countable - this is a good exercise (HINT: the set of finite strings from a countable set is countable).



            With this in hand, if $G$ is an uncountable group we can build an uncountable chain of subgroups of $G$, as follows:




            • We will define a countable subgroup $A_delta$ for every countable ordinal $delta$. There are uncountably many of these, so if we can do this we'll be done.


            • We let $A_0$ be the trivial subgroup.


            • Having defined $A_eta$ for every $eta<delta$, we let $a$ be some element of $G$ not in $bigcup_{eta<delta}A_eta$ - which exists, since this is a countable union of countable subgroups, and $G$ is uncountable - and let $A_delta=langle (bigcup_{eta<delta}A_eta)cup{a}rangle$.


            • It's easy to prove by transfinite induction that $(A_delta)_{delta<omega_1}$ is a strictly increasing chain of countable subgroups of $G$, so we're done.







            share|cite|improve this answer





















            • Both answers use the axiom of choice. In ZF, can there be an uncountable group with only a countable number of subgroups?
              – bof
              54 mins ago













            up vote
            4
            down vote










            up vote
            4
            down vote









            EDIT: bof's answer is the right one, but the construction below - while completely pointless overkill - is still an example of a useful technique, so I'm leaving this answer up.





            No, this cannot happen.



            Suppose $G$ is a group and $A$ is a countable subset of $G$. Then the closure of $A$ under the group operations ($*$ and $^{-1}$) of $G$, $langle Arangle$, is again countable - this is a good exercise (HINT: the set of finite strings from a countable set is countable).



            With this in hand, if $G$ is an uncountable group we can build an uncountable chain of subgroups of $G$, as follows:




            • We will define a countable subgroup $A_delta$ for every countable ordinal $delta$. There are uncountably many of these, so if we can do this we'll be done.


            • We let $A_0$ be the trivial subgroup.


            • Having defined $A_eta$ for every $eta<delta$, we let $a$ be some element of $G$ not in $bigcup_{eta<delta}A_eta$ - which exists, since this is a countable union of countable subgroups, and $G$ is uncountable - and let $A_delta=langle (bigcup_{eta<delta}A_eta)cup{a}rangle$.


            • It's easy to prove by transfinite induction that $(A_delta)_{delta<omega_1}$ is a strictly increasing chain of countable subgroups of $G$, so we're done.







            share|cite|improve this answer












            EDIT: bof's answer is the right one, but the construction below - while completely pointless overkill - is still an example of a useful technique, so I'm leaving this answer up.





            No, this cannot happen.



            Suppose $G$ is a group and $A$ is a countable subset of $G$. Then the closure of $A$ under the group operations ($*$ and $^{-1}$) of $G$, $langle Arangle$, is again countable - this is a good exercise (HINT: the set of finite strings from a countable set is countable).



            With this in hand, if $G$ is an uncountable group we can build an uncountable chain of subgroups of $G$, as follows:




            • We will define a countable subgroup $A_delta$ for every countable ordinal $delta$. There are uncountably many of these, so if we can do this we'll be done.


            • We let $A_0$ be the trivial subgroup.


            • Having defined $A_eta$ for every $eta<delta$, we let $a$ be some element of $G$ not in $bigcup_{eta<delta}A_eta$ - which exists, since this is a countable union of countable subgroups, and $G$ is uncountable - and let $A_delta=langle (bigcup_{eta<delta}A_eta)cup{a}rangle$.


            • It's easy to prove by transfinite induction that $(A_delta)_{delta<omega_1}$ is a strictly increasing chain of countable subgroups of $G$, so we're done.








            share|cite|improve this answer












            share|cite|improve this answer



            share|cite|improve this answer










            answered 1 hour ago









            Noah Schweber

            119k10146278




            119k10146278












            • Both answers use the axiom of choice. In ZF, can there be an uncountable group with only a countable number of subgroups?
              – bof
              54 mins ago


















            • Both answers use the axiom of choice. In ZF, can there be an uncountable group with only a countable number of subgroups?
              – bof
              54 mins ago
















            Both answers use the axiom of choice. In ZF, can there be an uncountable group with only a countable number of subgroups?
            – bof
            54 mins ago




            Both answers use the axiom of choice. In ZF, can there be an uncountable group with only a countable number of subgroups?
            – bof
            54 mins ago


















            draft saved

            draft discarded




















































            Thanks for contributing an answer to Mathematics Stack Exchange!


            • Please be sure to answer the question. Provide details and share your research!

            But avoid



            • Asking for help, clarification, or responding to other answers.

            • Making statements based on opinion; back them up with references or personal experience.


            Use MathJax to format equations. MathJax reference.


            To learn more, see our tips on writing great answers.





            Some of your past answers have not been well-received, and you're in danger of being blocked from answering.


            Please pay close attention to the following guidance:


            • Please be sure to answer the question. Provide details and share your research!

            But avoid



            • Asking for help, clarification, or responding to other answers.

            • Making statements based on opinion; back them up with references or personal experience.


            To learn more, see our tips on writing great answers.




            draft saved


            draft discarded














            StackExchange.ready(
            function () {
            StackExchange.openid.initPostLogin('.new-post-login', 'https%3a%2f%2fmath.stackexchange.com%2fquestions%2f3031937%2fcan-an-uncountable-group-have-a-countable-number-of-subgroups%23new-answer', 'question_page');
            }
            );

            Post as a guest















            Required, but never shown





















































            Required, but never shown














            Required, but never shown












            Required, but never shown







            Required, but never shown

































            Required, but never shown














            Required, but never shown












            Required, but never shown







            Required, but never shown







            Popular posts from this blog

            サソリ

            広島県道265号伴広島線

            Accessing regular linux commands in Huawei's Dopra Linux